Những câu hỏi liên quan
Hà Khánh Ngân
Xem chi tiết
Mai Tiến Đỗ
Xem chi tiết
Akai Haruma
2 tháng 1 2021 lúc 15:30

Mình nghĩ phần phân thức là $3x+3y+2z$ thay vì $3x+3y+3z$. Nếu là vậy thì bạn tham khảo lời giải tại link sau:

Cho x, y, z là các số thực dương thỏa mãn đẳng thức xy yz zx=5. Tìm GTNN của biểu thức \(P=\frac{3x 3y 2z}{\sqrt{6\left(... - Hoc24

Bình luận (2)
Nguyễn Đức Duy
Xem chi tiết
Lê Tài Bảo Châu
Xem chi tiết
Diệp Nguyễn Thị Huyền
Xem chi tiết
VUX NA
Xem chi tiết
Nguyễn Việt Lâm
7 tháng 8 2021 lúc 15:04

\(T\ge\dfrac{\left(x+y+z\right)^2}{x+y+z+\sqrt{xy}+\sqrt{yz}+\sqrt{zx}}\ge\dfrac{\left(x+y+z\right)^2}{x+y+z+x+y+z}=\dfrac{x+y+z}{2}\ge\dfrac{2019}{2}\)

Bình luận (0)
ミ★ήɠọς τɾίếτ★彡
7 tháng 8 2021 lúc 15:31

áp dụng BĐT:\(\dfrac{a^2}{x}+\dfrac{b^2}{y}+\dfrac{c^2}{z}\) với a,b,c,x,y,z là số dương

ta có BĐT Bunhiacopxki cho 3 bộ số:\(\left(\dfrac{a}{\sqrt{x}};\sqrt{x}\right);\left(\dfrac{b}{\sqrt{y}};\sqrt{y}\right);\left(\dfrac{c}{\sqrt{z}};\sqrt{z}\right)\)

ta có :

\(\dfrac{a^2}{x}+\dfrac{b^2}{y}+\dfrac{c^2}{z}\left(x+y+z\right)\)\(=\left[\left(\dfrac{a}{\sqrt{x}}\right)^2+\left(\dfrac{b}{\sqrt{y}}\right)^2+\left(\dfrac{c}{\sqrt{z}}\right)^2\right]\).\(\left[\left(\sqrt{x}\right)^2+\left(\sqrt{y}\right)^2+\left(\sqrt{z}\right)^2\right]\)\(\ge\left(\dfrac{a}{\sqrt{x}}.\sqrt{x}+\dfrac{b}{\sqrt{y}}.\sqrt{y}+\dfrac{c}{\sqrt{z}}.\sqrt{z}\right)^2=\left(a+b+c\right)^2\)

lúc đó ta có :\(\dfrac{a^2}{x}+\dfrac{b^2}{y}+\dfrac{c^2}{z}\ge\dfrac{\left(a+b+c\right)^2}{x+y+z}\)

ta có \(T=\dfrac{x^2}{x+\sqrt{yz}}+\dfrac{y^2}{y+\sqrt{zx}}+\dfrac{z^2}{z+\sqrt{xy}}\)\(\ge\dfrac{\left(x+y+z\right)^2}{x+\sqrt{yz}+y+\sqrt{zx}+z+\sqrt{xy}}\) mà ta có :

\(\sqrt{yz}+\sqrt{zx}+\sqrt{xy}\)\(\le\dfrac{x+y}{2}+\dfrac{x+z}{2}+\dfrac{z+y}{2}\)\(\Rightarrow\sqrt{yz}+\sqrt{zx}+\sqrt{xy}\le x+y+z\)

\(\Rightarrow T=\dfrac{2019}{2}\Leftrightarrow x=y=z=673\)

vậy \(\text{MinT}=\dfrac{2019}{2}\) khi và chỉ khi x=y=z=673

Bình luận (0)
Phạm Ngọc Thảo Ly
Xem chi tiết
phamthibaongoc
10 tháng 5 2016 lúc 20:15

khó quá!!!!!!!!!!!

Bình luận (0)
Đặng Thị Thu Hiền
Xem chi tiết
Trần Thị Loan
28 tháng 12 2014 lúc 9:38

Áp dụng BĐT Cô - si cho 2 số \(\frac{xy}{z};\frac{yz}{x}\)dương ta có: \(\frac{xy}{z}+\frac{yz}{x}\ge2\sqrt{\frac{xy}{z}.\frac{yz}{x}}=2\sqrt{y^2}=2y\)(1)

Tương tự. \(\frac{yz}{x}+\frac{zx}{y}\ge2\sqrt{\frac{yz}{x}.\frac{zx}{y}}=2\sqrt{z^2}=2z\) (2);

\(\frac{xy}{z}+\frac{zx}{y}\ge2\sqrt{\frac{xy}{z}.\frac{zx}{y}}=2\sqrt{x^2}=2x\)(3)

Cộng từng vế của (1)(2)(3) ta được \(2.\left(\frac{xy}{z}+\frac{yz}{x}+\frac{zx}{y}\right)\ge2\left(x+y+z\right)=2\Rightarrow P\ge1\)

Vậy Min P = 1 tại x= y = z = 1/3

Bình luận (0)
Best zanis
Xem chi tiết
Akai Haruma
25 tháng 5 2021 lúc 1:39

Lời giải:

Áp dụng BĐT AM-GM:

\(\frac{x^2}{2}+8y^2\geq 4xy\)

\(\frac{x^2}{2}+8z^2\geq 4xz\)

\(2(y^2+z^2)\geq 4yz\)

\(4y^2+1\geq 4y\)

\(4y+2\geq 4\sqrt{2y}\)

Cộng theo vế các BĐT trên ta có:

\(P+3\geq 4(xy+yz+xz)=\frac{9}{4}.4=9\Rightarrow P\geq 6\)

Vậy $P_{\min}=6$. Giá trị này đạt tại $(x,y,z)=(2,\frac{1}{2}, \frac{1}{2})$

Bình luận (2)